LSAT and Law School Admissions Forum

Get expert LSAT preparation and law school admissions advice from PowerScore Test Preparation.

 Administrator
PowerScore Staff
  • PowerScore Staff
  • Posts: 8950
  • Joined: Feb 02, 2011
|
#32450
Complete Question Explanation

Justify the Conclusion—SN, FIB. The correct answer choice is (E)

The section begins with a simple question that allows us a precise prephrase for a definitive answer choice, a very traditional beginning to a Logical Reasoning section.

The first two sentences provide us with important background information. Drawing from real life, the stimulus tells us that, in a certain country, a government agency took over most of that country’s large banks, which had failed. Now, several years later, the agency is looking to sell the banks, with the goal of strengthening the banking system.

Next, we are given the conditional rule applied by the stimulus author to reach the conclusion. If the former owners of the banks, those who permitted the banks to fail, buy them back now, then the banking system will not be strengthened. We can diagram this rule as:


FOBB = former owners buy the banks
S = banking system will be strengthened

Sufficient ..... ..... Necessary

FOBB ..... :arrow: ..... S


Based on the application of this rule, the author concludes it is unlikely that the agency will accomplish its goal of strengthening the banking system. This conclusion reflects the necessary condition of the conditional rule discussed above (S).

No premise is given to supply the fact to which the rule is applied to reach the conclusion. In its place is a blank space. Since the conclusion resulted from the application of the conditional rule, the missing premise will supply the sufficient condition of that rule, that the former owners of the banks will buy them back (FOBB).

The question stem identifies this as a Justify the Conclusion—Fill in the Blank question. Because the role of this answer choice is to prove the conclusion is valid, and not merely strengthen it, our prephrase is that the correct answer choice will state definitively that the former owners of the banks will buy them back, proving that the agency is unlikely to achieve its goal of strengthening the banking system.

Answer choice (A): The only way the author told us we could know the banking system will not be strengthened is for the former owners to buy back the banks. We have no reason to think that the agency’s inability to sell some—meaning “at least one, but possibly all”—of the banks would prevent it from strengthening the banking system. Also, be wary of weak language, such as the word “may,” in a Justify the Conclusion answer choice, which is designed to provide definitive proof of the conclusion, without gray area.

Answer choice (B): Since we do not know the identity of the single company that could buy more than one of the banks, this answer choice does not prove the conclusion is valid. To be correct, this answer choice would need to both identify the company as a former owner of the banks it purchased, and provide stronger language than a mere possibility that this could occur.

Answer choice (C): The current state of the economy, as opposed to the banking system, is irrelevant to the conclusion.

Answer choice (D):The comparison made here, between the financial strength of the banks being sold by the agency and the country’s other banks, is not relevant to the conclusion, nor is the implication that any strengthening of the banks being sold will take some unspecified period of time to occur. The conclusion focused on the strengthening of the banking system, rather than the strength of the banks being sold as compared to the strength of other banks within the system. And, the author did not commit to any time frame within which the strengthening must be accomplished.

Answer choice (E): This is the correct answer choice, because it definitively points us to the sufficient condition of the rule applied by the author, that the former owners of the banks buy them back. Since all of the bidders are the banks’ former owners, then should the banks be sold, and sold only to those who bid, then the sufficient condition would be satisfied. Recall that the conclusion hedged, by saying the agency is unlikely to meet its goal. The hedge is appropriate, because we are not told that the banks must be sold, nor that they must be sold only to the bidders. Given the identity of the bidders, the agency may take some different action.

Get the most out of your LSAT Prep Plus subscription.

Analyze and track your performance with our Testing and Analytics Package.